2025 AMC 12A Problems/Problem 19
Problem
Let
,
, and
be the roots of the polynomial
. What is the sum
Solution 1
We begin by factoring:
\begin{equation}
\begin{split}
&a^3b^2+a^2b^3+b^3c^2+b^2c^3+c^3a^2+c^2a^3 \\
&=a^2b^2(a+b)+b^2c^2(b+c)+c^2a^2(c+a)\\
&=(a^2b^2+b^2c^2+c^2a^2)(a+b+c) - a^2b^2c-b^2c^2a-c^2a^2b\\
&=(a^2b^2+b^2c^2+c^2a^2)(a+b+c) - (abc)(ab+bc+ca).\\
\end{split}
\end{equation}
From Vieta's Formulas, we know that
,
, and
. Therefore, the answer equals
~lprado
Solution 2
Notice that this is
.
By Vieta's (just like in solution 1), we have
, so
,
, and
, so this is equivalent to
by Vieta's.
~ScoutViolet
Solution 3 (BS)
Let’s just pick a nice and convenient a, b, c. Observe that
, so we can just have
). So expanding
we get
. Evaluating
, or
, we get
, so the answer is
. (i did this on the actual test)
~benjamintontungtungtungsahur
who is going to think of this on the test bro 😭😭 and change ur username dawg 🥀🥀
Solution 4 (Vieta's)
First, note that
,
,
. We can factor the original expression into:
~ABC09090927
Solution 5 (Newton's Sums) (If you suck at factoring like me)
Note that
By Newton's sums, we have:
Note that
From Vieta's formulas, we have
and
Solving, we get:
Therefore,
~ku220
Solution 6
Llike in solution 1, we begin by factoring:
\begin{equation}
\begin{split}
&a^3b^2+a^2b^3+b^3c^2+b^2c^3+c^3a^2+c^2a^3 \\
&=a^2b^2(a+b)+b^2c^2(b+c)+c^2a^2(c+a)\\
\end{split}
\end{equation}
From Vieta's Formulas, we know that
,
, and
.
square
to get
, then isolating one variable each time yields:
\begin{equation} \begin{split} &=-\frac{1}{c^2}(a+b)-\frac{1}{a^2}(b+c)-\frac{1}{b^2}(c+a)\\ \end{split} \end{equation}
Isolating a + b + c = 0, you get
\begin{equation} \begin{split} &=-\frac{1}{c^2}(-c)-\frac{1}{a^2}(-a)-\frac{1}{b^2}(-b)\\ &=-(\frac{ab+bc+ac}{abc})\\ \end{split} \end{equation}
This simplifies to:
\begin{equation} \begin{split} &=-(\frac{k}{-1})\\ &=k \end{split} \end{equation}
Therefore, the answer equals =
~BOTNATE
Solution 7 (Vieta + Factorization)
We can easily factor the expression into
By Vieta, we get
, so we can rewrite the expression into
which is just
. By Vieta we also get
and
. Therefore the expression is just
~backtosq-1
Video Solution 1 by OmegaLearn
Video Solution 2 by StressedPineapple
https://youtube.com/watch?v=NWBPm3lThH4&t=319s
Video Solution 3 by SpreadTheMathLove
https://www.youtube.com/watch?v=xipj4q--LCk
See Also
| 2025 AMC 12A (Problems • Answer Key • Resources) | |
| Preceded by Problem 18 |
Followed by Problem 20 |
| 1 • 2 • 3 • 4 • 5 • 6 • 7 • 8 • 9 • 10 • 11 • 12 • 13 • 14 • 15 • 16 • 17 • 18 • 19 • 20 • 21 • 22 • 23 • 24 • 25 | |
| All AMC 12 Problems and Solutions | |
These problems are copyrighted © by the Mathematical Association of America.